LSAT and Law School Admissions Forum

Get expert LSAT preparation and law school admissions advice from PowerScore Test Preparation.

 Administrator
PowerScore Staff
  • PowerScore Staff
  • Posts: 8919
  • Joined: Feb 02, 2011
|
#26775
(The complete setup for this game can be found here: lsat/viewtopic.php?t=11228)

Please post below with any questions!
User avatar
 baxleyce
  • Posts: 8
  • Joined: Apr 30, 2021
|
#87407
What is a quick way to solve this problem? I had trouble with this one as I had to diagram each answer option. Thanks in advance!
 Robert Carroll
PowerScore Staff
  • PowerScore Staff
  • Posts: 1787
  • Joined: Dec 06, 2013
|
#87420
baxley,

Because every answer choice leaves two people out, my thought was "Who, if left out, will cause the most problems?" Two variables strike me as tough ones to leave out: T, because if T is out, S is out (contrapositive of the second rule), and X (because X is a random, so is nice to have in, and because of the inference that T or X, or both, must always be in. That inference is necessary for Question #4, so I think it's fair to talk about it for Question #5!).

Looking at the answers, answer choice (B) puts T out, while answer choice (C) puts X out, and each also puts Q out. So these two answers look very good to start with, and, even if I did answer choice (C) first, I'd do answer choice (B) next, and be done after two diagrams. If you did answer choice (B) first (why not? it's alphabetically first!), then you'd do one diagram to prove it's right.

Robert Carroll
 concrottrox11@gmail.com
  • Posts: 29
  • Joined: Dec 07, 2021
|
#93142
Can someone please write out how they would solve this problem after picking the T and X to test out? I'm confused if the W/V and the Q/R have to be in both the in and out groups when setting up diagrams, particularly as it pertains to answer choices B and C.

Thank you.
 Adam Tyson
PowerScore Staff
  • PowerScore Staff
  • Posts: 5153
  • Joined: Apr 14, 2011
|
#93159
I actually wouldn't test anything to answer this question, concrottrox11@gmail.com, because you only need to refer to the work you already did on the previous questions! Here's what I mean:

Answer A is Q and S both being out. In my diagrams for questions 2, 3, and 4, it was possible for Q and S to both be out every time. That means putting them both out still leaves a lot of ways still available to solve the game, so it's a loser.

Answer B is attractive not only because it mentions T (and I agree with Robert, we should be very focused on T as a trigger here because it will, at a minimum, force S out as well), but also because those two were only both out in my local diagram for question 4. Contender!

Answer C is a loser because it was possible to have Q and X both out in the local scenarios for both question 2 and question 3, so putting them both out doesn't fully determine the solution.

Answer D is a loser because R and W could both be out in my diagrams for questions 3 and 4, so again, that means there is more than one solution when they are both out.

Finally, Answer E is wrong because I have scenarios from the local diagrams for question 2 and also question 4 in which R and V are both out, so I know there is more than one solution with them out.

Answer B must be the correct answer because I can see from my prior work that the other four are all losers. Use your prior work and save yourself the trouble of doing the work over again! In this case you could answer this question in under 10 seconds just by looking at your prior work on your scratch paper.

Now, with that said, if for some reason I completely forgot that I had already done a bunch of work that I could rely on to help me here, or if I hadn't done all that work on the prior questions (which you really should have done), here's how I would attack those two answer choices step by step:

Answer B:

1. Do what the question says and put Q and T out:
_ _ _ | QT _ _

2. S goes out because of T:
_ _ _ | QTS _

3. I think about putting W in, but I cannot, because then R and V have to go out, which they can't because there is only that one open space left in the Out group. Thus, W is out:
_ _ _ | QTSW

4. The three remaining variables must be in, and R can only be a leader if it's in, so that fixes the rest of the solution:
RXV|QTSW

Game solved, we have a winner!

Answer C (I would NOT test it if I had already tested answer B, but if for some reason I had done this one first, this is what the process would look like):

1. Do what the question says and put Q and X out:
_ _ _ | QX _ _

2. Hmmm.

3. Okay, if I put T out, S would also be out, but then I have a problem with W and V and R, so T must be in:
T _ _ | QX _ _
(Don't know if T is the leader - maybe yes, maybe no, I'll figure that out later)

4. I could put W in, which would push R and V out, and then S would be in. Do I have to? Maybe try putting W out instead?
T _ _ | QXW _

At this point it looks like any combo of RSV could be the last two in, and like I already determined, W might also be part of the In group, so this clearly is NOT solving the game. This answer is a loser, and I am picking answer B!

You can approach this question with the intention of doing a lot of work testing answer choices, but instead, approach it by looking for a simpler, easier way to get the job done, and that is using the work you already did. Saves time, saves effort, saves space on your scratch paper, and gets you the right answer in no time flat!

Get the most out of your LSAT Prep Plus subscription.

Analyze and track your performance with our Testing and Analytics Package.